留学生课程辅导

AP微观经济考试真题及考情分析

文章发布时间:2023-02-16 10:37:14文章来源:考而思在线阅读量:216

2023年AP考试难度提高了?从2021年北美AP微观经济考情及考试真题来看。CB放出的这两套试卷来,整体难度中等偏上,既有以往典型的市场结构考题,也有新出现的知识点融合以及更偏应用的情景题。几道大题中所覆盖的知识点与大家练习过的往年真题并无太大差异,主要包括垄断的图像,用边际分析解释企业达到利润最大化的选择等。

但与此同时,CB对于大题最后一至两小问的难度也有了一定的提升,例如往年真题通常一整道题会考一个较为完整的知识点,但set1和set2的Q1都融合了不同的市场结构,可能会让同学们在刚接触时感到略微陌生和紧张。

与往年题目对比的点评

整体上来讲,Q1与2018和2019的并无太大差距,问题基本围绕monopoly的图像展开,以及相关政策管制给市场带来的变化。不过正如前文所写,这次考试的题目安排上有一个较为明显的改变就是在一道题目中融合了不同的市场结构和变化,例如set1Q1就是从垄断变成了一个更加具有竞争性的结构,而set2Q1则是从单一垄断变成了寡头垄断,考察了博弈论的内容(以往会单独出一道题)那么这实际上是把FRQ的难度稍稍提高了一些,需要同学们找准题目中市场结构的变化。

两套试卷的Q2在知识点考察上并无太大的特殊之处,但题目叙述比之前略微复杂一些,更加偏向情景描述,需要同学们从字里行间提取信息,仔细分析。set1Q2考到了企业在要素市场中雇佣劳动力的知识点,最后一问的变化则是当企业需要给员工购买保险时,员工工资的变化,较为新颖。set2Q2则是考察的外部性的图像以及政策管制,也是需要从题干中提炼信息并分析。

Q3都可算是比较常规性的题目,与以往考题较为相似。set1Q3考察的是价格地板(价格下限)和市场变化,set2Q3则是边际分析以及一道市场结构的辨析题,除最后一小问以往并未考到过,其余部分应该都在同学们的刷题范围之内。

AP微观经济考试真题及考情分析

逐题分析

Set1:

Q1:(a)

The government of an island nation grants SkyRunner Airlines exclusive monopoly rights to serve the islandSkyRunner is earning positive economic profits.

(a) Draw a correctly labeled graph for SkyRunner, and show each of the following.() The profit-maximizing quantity of tickets, labeled Qm(ii) The profit-maximizing price of a ticket, labeled Py(iii) The area representing profits, shaded completely

考点:unit 4-monopoly的图像

分析:题目中说政府现在给了SkyRunner Airlines垄断的权力,并且该企业有正向的经济利润,要求画图。首先标注横轴为产量,纵轴为价格,由垄断可得知企业的demand curve是向下倾斜的,并且MR低于demand;接着画一条斜向上倾斜的MC,MC和MR的交点对应的横轴量就是利润最大化产量,从该产量对应到需求曲线上的点,该点的纵轴数值就是利润最大化的价格。最后,在画ATC曲线时需注意其与当前价格的关系,因为此时企业是有经济利润的,说明此时的价格高于ATC,而ATC又会被MC穿过最低点,明确这几点之后画图,并标注(P-ATC)*Q,也就是价格和ATC之间的长方形面积就是经济利润。

Q1:(b)

Now the island's tourist bureau asks the government to consider the following two proposals.

 Proposal I: Set a price ceiling on tickets that eliminates all deadweight loss.Proposal Il: Eliminate SkyRunner's monopoly rights, which will remove all barriers to entry.(b) Suppose the government adopts proposal I. On your graph in part (a), indicate the quantity of tickets sold inthe short run, labeled Oc.

考点:unit 2-price ceiling&unit 4-deadweight loss in monopolies

分析:该政策是想通过设置价格天花板来消除全部的无谓损失,也就是让P=MC,所以在demand和MC交点对应的横轴数值就是Qc。

Q1:(c)

(c) Suppose instead the government adopts proposal Il. How will each of the following be affected in the longrun compared to the market conditions in part (a) ?

(i) The quantity of tickets sold by SkyRunner. Explain.(ii) The price elasticity of demand for SkyRunner's airline service. Explain.(iii) SkyRunner's profits(iv) The deadweight loss in the market. Explain.

考点:unit 3&4-different market structures

分析:当政府移除了SkyRunner的垄断权之后,该企业正向的经济利润会吸引更多企业进入这个市场,市场的竞争性增强,而对于消费者来说,相近的替代品变得更多,所以整体上对于SkyRunner产品的需求价格弹性变大,该企业利润减少,市场的无谓损失减少。

Q2:(a)

2Assume that Schmitt Inc. provides car parking services in a perfectly competitive output market and hires lahorin a perfectly competitive input market. The market price per car parked is $10, the daily market wage perworker is $100, and fixed costs are $50 per day. The table above shows the number of workers required topark different quantities of cars per day.

(a) Calculute the marginal revenue product of the second worker. Show your work.

考点:unit 5-marginal revenue product of labor(MRPL)

分析:根据大体我们可以得知当前这家企业的产品和要素市场均为完全竞争的结构,停车的市场价格,也就是产品价格是$10,员工均衡工资为$100,固定成本是$50。题目中给出了雇佣不同员工数的总产量,让求第二个员工的MRP。MRP=price of product*marginal product of labor,题目中给出了price of product=$10,而marginal product of the second labor可以从表格中计算得出,二者相乘即可。

Q2:(b)

(b) How many workers will Schmitt Inc. hire to maximize profit? Relative to this number of hired workerscxplain why Schmitt Inc. will not hire onc additional worker. Your answer must use marginal analysis andnumbers from the table.

考点:unit 5-hiring labor in a competitive market

分析:企业为了做到利润最大化会在雇佣员工时尽量达到MRP=MFC,而完全竞争市场当中工资由市场决定MFC=wage,所以最优情况是MRP=wage,如果没有完全相等的情况则停在最后一名员工的MRP>wage的地方,因为此时企业雇佣该员工还能赚取正向利润,但如果MRP$100,而MRP of the fifth labor=9*10=$90<$100,所以企业不会雇佣第五个员工。

Q2:(c)

(c) Calculate the daily profit for Schmitt Inc. at the profit-maximizing quantity identified in part (b).Show your work

考点:unit 3-利润的计算

分析:根据前一问可得知此时企业会雇佣4个劳动力,所以总收入TR=P*Q=10*45=$450,而总成本TC=VC+FC=4*100+50=$450,利润等于TR-TC=$0。

Q3:(a)

The diagram above shows the market for corn in the country of Mlicroland. Corn is produced and sold in a

constant-cost, perfectly competitive market.

(a) Calculate the total revenuc carned by corn farmers at the market cquilibrium price. Show your work.

考点:unit 3-总收入的计算

分析:总收入TR=P*Q=5*50=$250.

Q3:(b)

(b) In an attempt to assist corn farmers in Microland, the government sets a $7 price floor on corn.

(i) How many bushels of corn will be exchanged at the price floor?

(ii) Calculate the deadweight loss associated with the price floor. Show your work.

(iii) Assume the government agrees to buy the unsold quantity at S7. Calculate the producer surplus.Show your work.

(iv) Assume the price floor and the government buying program remain in cffect. In addition, assumcthe demand for corn docs not changc. In the long run, will the quantity of corn purchased by thegovernment increase, decrease, or remain the same? Explain.

考点:unit 2&3-price control,producer surplus,deadweightloss

分析:当政府设置$7的价格地板时,供给量是70,需求量时30,最终的成交量是30;无谓损失可以用(7-3)*(50-30)/2算出;而当政府购买全部多余产品时,生产者剩余是价格线之下,供给线之上围起来的全部面积,用7*70/2可以算出。

Set2:

Q1:(a)

1. NCHart is a corporation that has developed and patented a new drug to treat heart disease. There are nosubstitutes for this drug, giving NCHart a monopoly.

(a) Draw a correctly labeled graph of NCHart making a positive economic profit, and show each of thefollowing.

(i) The profit-maximizing quantity, labeled Qm

(ii) The profit-maximizing pricc, labcled P

考点:unit 4-monopoly的图像

分析:题目中说NCHart对某种药物有垄断的控制,并且该企业有正向的经济利润,要求画图。首先标注横轴为产量,纵轴为价格,由垄断可得知企业的demand curve是向下倾斜的,并且MR低于demand;接着画一条斜向上倾斜的MC,MC和MR的交点对应的横轴量就是利润最大化产量,从该产量对应到需求曲线上的点,该点的纵轴数值就是利润最大化的价格。最后,在画ATC曲线时需注意其与当前价格的关系,因为此时企业是有经济利润的,说明此时的价格高于ATC,而ATC又会被MC穿过最低点,明确这几点之后画出ATC曲线即可。

Q1:(b)

(b) At Qm from part (a)(), is demand elastic, unit elastic, or inelastic? Explain using information from the

graph.

考点:unit 4-垄断企业运行的区间和弹性

分析:由于此时企业的产量是利润最大化的,MR>0,而MR与横轴的交点位于demand和横轴焦点的中点,所以此时对应的需求曲线是中点的左上半部分,是富有弹性的。

Q1:(c)

(c) Instead of maximizing profit, suppose NCHat considers providing the new drug to as many patients aspossible as long as it can generate enough revenue to cover its total costs.

(i) On your graph from part (a), show the quantity that is consistent with this goal, labeled Qz

(ii) At O, from part (c)

(i), is there a deadweight loss? Explain.

考点:unit 3&4-deadweight loss and breakeven point

分析:题目中说现在企业不再追求利润最大化,而是尽量多生产直至TR=TC,也就是让P=ATC,对应到图像中就是demand和ATC的交点,其对应的横轴数值就是Qz;而此时MC是高于价格的,产量比社会最优产量还要多,总剩余大于初始状态,不会有无谓损失。

Q1:(d)

(d) NCHart's patent expires next year, and a new firm, TXDrug, is considering whether to Enter this market orStay Out. NCHart can either produce Qm or Q,. The firms independently and simultaneously choose theiractions. The first entry in the payoff matrix represents NCHart's payoff and the second entry representsTXDrug's payoff. Both firms have complete information. Use the payoff matrix below to answer the followingquestions.

(i) Does TXDrug have a dominant strategy? Explain using strategies and payoffs from the payoffmatrix.

(ii) What is the best response for NCHart if TXDrug chooses to Stay Out?

(iii) Identify the Nash equilibrium.

考点:unit 4-game theory in oligopolies

分析:当NCHart选择Qm时,TXDrug stay out利润为$0,enter利润为$1,$1>$0,TXdrug会选择enter;当NCHart选择Qz时,TXDrug stay out利润为$0,enter利润为-$1,$0>-$1,TXdrug会选择stay out,所以TXDrug的选择是随对方的选择变化的,TXDrug没有占优策略。第二小问,当TXDrug stay out时,NCHart选Qm利润为$10,Qz利润为$0,$10>$0,所以NCHart会选择制定Qm。根据前两问我们可以得知TXDrug没有占优策略,而NCHart再经过对比之后可以得知是有占有策略的,它会始终选择指定Qm,所以那什均衡就是NCHart制定Qm,TXDrug选择enter。

Q2:(a)

2. Copper is produced in a perfectly competitive market with an upward-sloping supply curve and adownward-sloping demand curve. Assume the production of copper results in liquid waste, which seeps intolocal rivers. The contaminated river water causes human illncsses and crop failures downstream. The marginacxternal cost from producing copper is constant across all quantitics of copper produced.

(a) Draw a correctly labeled graph of the copper market with the marginal social benefit (MSB), marginalprivate benefit (MPB). marginal social cost (MSC). and marginal private cost (MPC) curves, and show cach ofthe following.

(i) The market equilibrium quantity, labeled Qm(ii) The socially efficient quantity. labeled Qs

考点:unit 6-negative externality的图像

分析:题干中说铜的生产会带来污染,所以该商品是有负外部性的,图像中有两条成本曲线,MSC>MPC,两条线都是斜向上倾斜的并且垂直差距恒定(marginal external cost is constant)题目中没有特殊说明benefit,所以MSB=MPB,两条线都是斜向下倾斜的。市场均衡数量是MPC和MPB的交点对应的横轴量,社会最优数量则是MSB和MSC的交点对应的横轴量;社会最优数量应小于市场均衡数量。

Q2:(b)

(b) Suppose the demand for copper decreases. On your graph in part (a), show the deadweight loss at the new

market equilibrium, shaded completely.

考点:unit 5-deadweight loss in the production of goods with negative externality

分析:这题告诉我们现在对于铜的需求增加了,所以MB会向右移动,导致市场均衡数量增加,新的市场均衡数量与新的社会最优数量之间所差的三角形即可代表无谓损失。

Q2:(c)

(c) Suppose the government is considering levying a tax on copper.

(i) What per-unit tax level would achieve the socially optimal quantity?

(ii) Explain why a lump-sum tax on producers will not achieve the socially optimal quantity in the shortrun.

考点:unit 6-government taxes

分析:此时如果为了达到社会最优数量,政府可以收取与边际外部成本(MEC)相等的单位税,可以使私人成本增加,MPC曲线向左(上)移动,直至MPC与MSC重合,市场均衡数量与社会最优产量相等。但lump-sum tax(定额税或一次总赋税)短期内仅改变固定成本,并不会影响MPC,所以并不能使数量变到与社会最优数量一致。

Q3:(a)

3. The table below shows the total cost and total benefit of adyertisements placed by AZY Foods. a firm in the retail food market.

(a) Calculate the total net benefit of placing three advertisements.

Show your work

考点:unit 1–cost and benefit analysis

分析:当企业达三个广告的时候,总收益是$3000,总成本是$800,净收益就用3000-800即可。

Q3:(b)

(b) Calculate the marginal net benefit of the third advertisement. Show your work

考点:unit 1-cost and benefit analysis

分析:这题要求的是第三个广告的边际净收益,需要用边际收益减边际成本;第三个广告的边际收益用3000-2200可以求得,边际成本用800-500可以求得,两者相减即可。

Q3:(c)

(c) What is the optimal number of advertisements placed by AZY Foods? Explain using marginal analysis.

考点:unit 1-cost and benefit analysis

分析:对于企业来讲,只要边际收益大于边际成本,打广告就还是有利可图的,所以AZY公司打第四个广告的边际收益是3600-3000=$600,边际成本是1300-800=$500,边际收益大于边际成本,所以第四个广告还会打;但是第五个广告的边际收益是4000-3600=$400,边际成本是2100-1300=$800,边际收益小于边际成本,AZY公司不会打第五个广告,最优的广告数量就是4个。

Q3:(d)

(d) Suppose over the next year the marginal benefit that AZY Foods receives from each advertisementincreases by $300. Identify the optimal number of advertisements.

考点:unit 1-cost and benefit analysis

分析:题目中说此时每个广告的边际收益都增加了$300,在变化之后,需要将新的MB和MC进行对比,步骤类似前一小问,最终可得出最优的广告数量是4个。

Q3:(e)

(C) There are many firms in the retail food market. Each firm places its own firm-specific advertisementswithout considering the actions of its competitors. In what market structure is AZY Foods operating?

考点:unit 3&4-different market structures

分析:假设现在这个市场中有很多家企业,每家企业都会打广告并且不考虑其他竞争对手的选择,问此时描述的是哪种市场结构。从很多家企业我们可以推断出该市场是具有竞争性的,而企业又会对各自的产品打广告则说明不同企业的产品是有区别的,所以推断出该市场应为垄断竞争monopolistic competition。

当前文章链接:

凡来源标注“考而思在线”均为考而思在线原创文章,版权均属考而思在线所有,任何媒体、网站或个人不得转载,否则追究法律

留学生辅导

学术资料

定制课程

电话咨询

400-640-8558

客服微信

在线客服